7
$\begingroup$

Quantum mechanics is equivalent with Feynman path ensemble, which after Wick rotation becomes Boltzmann path ensemble, which can be normalized into stochastic process as maximal entropy random walk (MERW).

But Boltzmann path ensemble has also spatial realization: 1D Ising model and its generalizations: Boltzmann distribution among spatial sequences of spins or some more complicated objects.

For $E_{uv}$ energy of interaction between $u$ and $v$ neighboring spins or something more general, define $M_{uv} = \exp(-\beta E_{uv})$ as transition matrix and find its dominant eigenvalue/vector: $M \psi = \lambda \psi$ for maximal $|\lambda|$. Now it is easy to find (e.g. derived here) that probability distribution of one and two neighboring values inside such sequence are:

$$Pr(u) = (\psi_u)^2\qquad\qquad Pr(u,v) = \psi_u \frac{M_{uv}}{\lambda} \psi_v$$

The former resembles QM Born rule, the latter TSVF – the two ending $\psi$ come from propagators from both infinities as $M^p \approx \lambda^p \psi \psi^T$ for unique dominant eigenvalue thanks to Frobenius-Perron theorem. We nicely see this Born rule coming from symmetry here: spatial in Ising, time in MERW.

Having Ising-like models as spatial realization of Boltzmann path integrals getting Born rule from symmetry, maybe we could construct Bell violation example with it?

Here is MERW construction (page 9 here) for violation of Mermin’s Pr(A=B) + Pr(A=C) + Pr(B=C) >= 1 inequality for 3 binary variables ABC, intuitively “tossing 3 coins, at least 2 are equal” (e.g. here is QM violation):

enter image description here

From Ising perspective, we need 1D lattice of 3 spins with constraints – allowing neighbors only accordingly to blue edges in above diagram, or some other e.g. just forbidding |000> and |111>.

Measurement of AB spins is defect in this lattice as above – fixing only the measured values. Assuming uniform probability distribution among all possible sequences, the red boxes have correspondingly 1/10, 4/10, 4/10, 1/10 probabilities – leading to Pr(A=B) + Pr(A=C) + Pr(B=C) = 0.6 violation.

Could this kind of spin lattice construction be realized?

What types of constraints/interaction in spin lattices can be realized?

While Ising-like models provide spatial realization of Boltzmann path integrals, is there spatial realization of Feynman path integrals?


Update: sketch of derivation of the formulas above of probabilities inside Ising's Boltzmann sequence ensemble:

enter image description here

$\endgroup$
3
  • 1
    $\begingroup$ Another paper about Bell violation with Ising model: journals.aps.org/prl/abstract/10.1103/PhysRevLett.123.170604 $\endgroup$
    – Jarek Duda
    Jan 16, 2020 at 12:10
  • $\begingroup$ If we can violate Bell-like inequalities with Ising, maybe we could realize quantum-like computers this way? While such Wick-rotated quantum gates seem computationally a bit weaker, such spatial realization allows to fix amplitudes from both directions: left and right, what seems to allow to quickly solve NP-complete problems from Boltzmann sequence ensemble - end of arxiv.org/pdf/1912.13300 $\endgroup$
    – Jarek Duda
    Jan 22, 2020 at 4:29
  • 1
    $\begingroup$ Lecture about MERW and such view on Ising model: youtu.be/k8T1jVBHWrM $\endgroup$
    – Jarek Duda
    Jun 15, 2020 at 10:46

0

Your Answer

By clicking “Post Your Answer”, you agree to our terms of service and acknowledge you have read our privacy policy.